Finding Area Between Two Polar Curves

  • Thread starter Thread starter Lancelot59
  • Start date Start date
  • Tags Tags
    Area Curves Polar
Click For Summary
SUMMARY

The discussion focuses on calculating the area between two polar curves: r=sqrt(3)cos(theta) and r=sin(theta). The user initially set up the integral correctly but encountered discrepancies with the final answer, which was attributed to integration errors. The correct area calculation involves integrating the squared functions over specified intervals, leading to the conclusion that the book's answer of 5pi/24 - sqrt(3)/4 is indeed correct, confirming that the user's setup was valid but the integration was flawed.

PREREQUISITES
  • Understanding of polar coordinates and curves
  • Knowledge of integration techniques in calculus
  • Familiarity with trigonometric identities
  • Ability to interpret and set up definite integrals
NEXT STEPS
  • Review polar area calculations in calculus
  • Practice integration of trigonometric functions
  • Study the properties of polar curves and their intersections
  • Explore common pitfalls in definite integral setups
USEFUL FOR

Students and educators in mathematics, particularly those studying calculus and polar coordinates, as well as anyone interested in enhancing their integration skills.

Lancelot59
Messages
640
Reaction score
1
This particular problem is just confusing me in the setup. I need to find the area that is inside both:
r=sqrt(3)cos(theta) and r=sin(theta)

It makes a petal type shape. I was beating my head around for a while, but I reasoned that since the equation used to find the area cuts out in a straight line. I could just move in either direction following the appropriate functions and get the area by adding the two parts together:

\frac{1}{2}[\int_{\frac{\pi}{3}}^{\frac{\pi}{2}} (\sqrt{3}cos(\theta))^{2}) d\theta + \int_{0}^{\frac{\pi}{3}} (sin(\theta)^{2} d\theta]

It makes sense to me, but my final answer was 17pi\4 + 3sqrt(3)/8, however the book states the answer is 5pi/24 - sqrt3/4. Is my setup wrong, or did I just mess up with the integration?
 
Last edited:
Physics news on Phys.org
You just messed up the integration.

It's probably just a typo, but the book's answer should be 5\pi/24-\sqrt{3}/4.
 
It was a typo. The book had that answer. Thanks, I'll check my work again.
 
Question: A clock's minute hand has length 4 and its hour hand has length 3. What is the distance between the tips at the moment when it is increasing most rapidly?(Putnam Exam Question) Answer: Making assumption that both the hands moves at constant angular velocities, the answer is ## \sqrt{7} .## But don't you think this assumption is somewhat doubtful and wrong?

Similar threads

  • · Replies 2 ·
Replies
2
Views
2K
  • · Replies 20 ·
Replies
20
Views
2K
  • · Replies 19 ·
Replies
19
Views
2K
  • · Replies 10 ·
Replies
10
Views
2K
  • · Replies 14 ·
Replies
14
Views
2K
Replies
3
Views
2K
  • · Replies 3 ·
Replies
3
Views
2K
  • · Replies 1 ·
Replies
1
Views
2K
  • · Replies 5 ·
Replies
5
Views
2K
  • · Replies 3 ·
Replies
3
Views
2K